what is the equation of the line that passes through (1,5) and (3,1)

Answers

Answer 1

Answer:

Below

Step-by-step explanation:

First let's find m

     m = Δy / Δx = y2 - y1 / x2 - x1

         = 1 - 5 / 3 - 1

         = -2

Now to find b, you can sub in the values of (x,y) for (1,5)

5 = (-2)(1) + b

-b = -2 - 5

-b = - 7

b = 7

Final equation

y = -2x + 7

Hope this helped!

Answer 2

Answer:

4x + 2y - 14

Step-by-step explanation:

I hope it is understandable.


Related Questions

convert 5.875baae 10 to base 2​

Answers

Yes exactly this helped sooo much
If you meant 5875 base 10 to base 2, it should be 1011011110011.

Help
Find the volume of this cone.
Use 3 for T.
6ft
V =
3
Tr2h
1
10ft
V
V-
[?]ft3
A

Answers

Answer:

[tex]V=90[/tex]

Step-by-step explanation:

Step 1:  Find the volume of the cone

[tex]V = \frac{\pi *r^{2}*h}{3}[/tex]

Plug in the values

[tex]V = \frac{\pi *3^{2}*10}{3}[/tex]

Simplify and Solve

[tex]V = \frac{\pi *9*10}{3}[/tex]

[tex]V = \frac{90\pi }{3}[/tex]

[tex]V = \frac{90*3 }{3}[/tex]

[tex]V=90[/tex]

Answer:  [tex]V=90[/tex]

Round 96.1825899389 to 4 decimal places.

Answers

96.1826

Because the 5th number is over 5 you round up the 4th number to 6 :)

the sum of the ages of chukwu and Caroline is 45 years if the difference in their age is 3 years find the age of chukwu (chukwu is older than Caroline)​

Answers

Step-by-step explanation:

let,the sum of the ages of chukwu & Caroline be X & Y.

now,

X+y=45

X-y=3

(+) (+) (-)

__________

2x=42

or,X=42/2

or,X=21

putting the value of X,

X+y=45

or, 21+y=45

or,y=45-21

or,y=24

therefore,X=21

y=24

What number is square and a cube.
E.g.
2²=4
3³=9
What number has a cube root AND a square root??

Answers

Answer:

2²=4 is square root and 3³=9 is cube root

Step-by-step explanation:

because 3×3×3=3³and 2×2=2²

64:
4x4x4 = 64
8x8 = 64

The data set represents the total number of pencils each student in a class needs to sharpen.

0, 1, 1, 1, 2, 3, 4, 4, 6, 6, 9

Which box plot correctly represents the data?

A number line goes from 0 to 10. The whiskers range from 0 to 9, and the box ranges from 2 to 6. A line divides the box at 3.
A number line goes from 0 to 10. The whiskers range from 0 to 9, and the box ranges from 2 to 6. A line divides the box at 4.
A number line goes from 0 to 10. The whiskers range from 0 to 9, and the box ranges from 1 to 6. A line divides the box at 4.
A number line goes from 0 to 10. The whiskers range from 0 to 9, and the box ranges from 1 to 6. A line divides the box at 3.

Answers

Given:

The data set is:

0, 1, 1, 1, 2, 3, 4, 4, 6, 6, 9

To find:

The correct statement that represents the box plot of the data correctly.

Solution:

We have,

0, 1, 1, 1, 2, 3, 4, 4, 6, 6, 9

Divide the data in 2 equal parts.

(0, 1, 1, 1, 2), 3, (4, 4, 6, 6, 9)

Divide each parenthesis in 2 equal parts.

(0, 1), 1, (1, 2), 3, (4, 4), 6, (6, 9)

Here,

Minimum value = 0

First quartile = 1

Median = 3

Third quartile = 6

Maximum value = 9

A number line goes from 0 to 10. The whiskers range from 0 to 9, and the box ranges from 1 to 6. A line divides the box at 3.

Therefore, the correct option is D.

Answer:

its d

Step-by-step explanation: my yt Tray Clayps

The telephone numbers in a small town have two digits. They run from 00 to 99.

Of the 100 possible numbers, those that become smaller when reversed are not used, i.e. 21 is not used.

What is the maximum number of telephone numbers this town could have?

Answers

Answer:

[tex]55[/tex]

Step-by-step explanation:

We'll be using case-work to solve this problem. Let's call the ones digit of the telephone number [tex]B[/tex] and the tens digit [tex]A[/tex]. Each telephone number can be represented as [tex]AB[/tex].

Since the question states that numbers that are smaller when their digits are reversed are not used, we have the following inequality:

[tex]B\geq A[/tex]

This is because if [tex]A>B[/tex], the number would become smaller when [tex]A[/tex] and [tex]B[/tex] are switched in [tex]AB[/tex]. However, if [tex]A=B[/tex] or [tex]B>A[/tex], the number will not become smaller.

Let's work our way up starting with [tex]A=0[/tex]. If [tex]A=0[/tex], there are 10 other numbers (0-9) that we can choose for [tex]B[/tex] that adhere to the condition [tex]B\geq A[/tex]:

[tex]0B,\\01, 02, 03,...[/tex]

Therefore, there are 10 possible telephone numbers when the tens digit is 0.

Repeat the process, now assigning [tex]A=1[/tex]. Now, we only have the digits 1-9 to choose from for [tex]B[/tex], since [tex]B[/tex] needs to be greater than or equal to A. Therefore, there are 9 possible telephone numbers when the tens digit is 1.

This pattern continues. As we work our way up through the cases (when increasing [tex]A[/tex] by 1), the number of possible telephone numbers decreases by 1, since there becomes one less option for [tex]B[/tex].

The last case would be [tex]A=9[/tex] in which case there would only be one option for [tex]B[/tex] and that would be 9.

Since there are 10 cases (0-9), add up the possible telephone numbers for each case:

[tex]\displaystyle \sum_{n=1}^{10}n=1+2+3+4+5+6+7+8+9+10=\boxed{55}[/tex]

Alternatively, recall that the sum of this series can be found using [tex]\frac{n(n+1)}{2}[/tex], where [tex]n[/tex] is the number of values in the set. In this case, [tex]n=10[/tex], and we have:

[tex]1+2+3+4+5+6+7+8+9+10=\frac{10(11)}{2}=\frac{110}{2}=\boxed{55}[/tex]

Which statement is true regarding the graphed functions
f(4)=g(4)

F(4)=g(2)

F(2)=g(-2)

F(-2)=g(-2)

Answers

3.
When you are on the other side of the = you are opposite.

Answers plssssssssssssssssss

Answers

Answer:

3/2

Step-by-step explanation:

if u divide two fractions, it is same as multiplying with its reciprocal

The shaded area in the distribution below represents approximately 95% of the data. Use the diagram to find the mean and the standard deviation.

Answers

Answer:

85 i guess

Step-by-step explanation:

The mean of the distribution is 70 and the standard deviation of the distribution is 4.

The shaded area in the distribution below represents approximately 95% of the data. This means that the shaded area is equal to 1.96 standard deviations from the mean.

The 68-95-99.7 rule states that 95% of the data in a normal distribution falls within 2 standard deviations of the mean. So, the shaded area in the distribution below represents the area between 1.96 standard deviations below the mean and 1.96 standard deviations above the mean.

The mean of the distribution is the center of the shaded area. The standard deviation of the distribution is the distance from the mean to the edge of the shaded area.

In this case, the mean of the distribution is approximately 70. The standard deviation of the distribution is approximately 4.

Here is a step-by-step explanation of how to find the mean and the standard deviation:

Identify the shaded area in the distribution.

Remember that the shaded area represents 95% of the data.

Use the 68-95-99.7 rule to determine that the shaded area is between 1.96 standard deviations below the mean and 1.96 standard deviations above the mean.

Find the mean by averaging the two points on the distribution that are 1.96 standard deviations from the mean.

Find the standard deviation by subtracting the mean from one of the points on the distribution that is 1.96 standard deviations from the mean.

Therefore, the mean of the distribution is approximately 70 and the standard deviation of the distribution is approximately 4.

To learn more about standard deviation here:

https://brainly.com/question/13498201

#SPJ2

A car left Sydney at 9 am and arrived in
Melbourne at 6 pm. The distance travelled
was 711 km. Find the car's average speed.
km/h

Answers

Answer:

average speed = distance/ time

711/9

79km/h

Drag the tiles to the correct boxes to complete the pairs. Not all tiles will be used. Match the estimated value of each expression with its position on the number line. + A B C D E ++ + 1 2 3 4 5 6 7 -2 -1 0 A B С D E V90-V40 V35 - 42 V5 – V6 2127-148 V54 - V24 18 – V8​

Answers

Answer:

1. D

2.C

3. E

4.A

Step-by-step explanation:

the guy above or below me is incorrect.

Which of the following equations of a line is NOT written in standard form?
A. 12x - 2y = 82
B. y = -2x - 1
C. x – 7y = 22
D. -x + 5y = 10

Answers

Answer: B

Step-by-step explanation:

Its written in slope intercept form not standard

A package of 4 red,white and blue hats costs $8. What is the unit rate

Answers

Answer:

$2/ hat

Step-by-step explanation:

Find the unit rate:

$ to hats

8 to 4

Divide by four to get the unit rate; over 1

2 to 1

So, $2 per hat

Hope this helps!

The answer is 2/ hat Bc I got it right

I NEED HELP WITH THIS!! 50 POINTS!!!

Drag each system of equations to the correct location on the table.
Classify each system of equations as having a single solution, no solution, or infinite solutions.

Answers

Answer:

Starting with the first one, we need to convert both of the equations into slope-intercept form. y = -2x + 5 is already in that form, now we just need to do it to 4x + 2y = 10.

2y = -4x + 10

y = -2x +5

Since both equations give the same line, the first one has infinite solutions.

Now onto the second one. Once again, the first step is to convert both of the equations into slope-intercept form.  

x = 26 - 3y becomes

3y = -x + 26

y = -1/3x + 26/3  

2x + 6y = 22 becomes

6y = -2x + 22

y = -1/3 x + 22/6

Since the slopes of these two lines are the same, that means that they are parallel, meaning that this one has no solutions.  

Now the third one. We do the same steps.  

5x + 4y = 6 becomes

4y = -5x + 6

y = -5/4x + 1.5

 

10x - 2y = 7 becomes

2y = 10x - 7

y = 5x - 3.5

Since these two equations are completely different, that means that this system has one solution.

Now the fourth one. We do the same steps again.  

x + 2y = 3 becomes

2y = -x + 3

y = -0.5x + 1.5

4x + 8y = 15 becomes

8y = -4x + 15

y = -1/2x + 15/8

Once again, since these two lines have the same slopes, that means that they are parallel, meaning that this one has no solutions.  

Now the fifth one.  

3x + 4y = 17 becomes

4y = -3x + 17

y = -3/4x + 17/4

-6x = 10y - 39 becomes

10y = -6x + 39

y = -3/5x + 3.9

Since these equations are completely different, there is a single solution.  

Last one!

x + 5y = 24 becomes

5y = -x + 24

y = -1/5x + 24/5

5x = 12 - y becomes

y = -5x +12

Since these equations are completely different, this system has a single solution.

Step-by-step explanation:

hope this helps you out:)

Find the Quotient. Do NOT round

22.14 divided by 1.2

Answers

Answer:

18.45

Step-by-step explanation:

I used a calculator

pls pls pls help meeeeee

Answers

Answer:

i think you just extend the coordinates to the side, except the right point, by 3, and then the bottom ones go down by 3, and the top one goes up by 3

Step-by-step explanation:

Which angle number represents an angle adjacent to /EHD?

Answers

Angle 5. To read the angle EHD you start at point E, then go to H, then to D. So that angle is marked 6. And 5 is adjacent (next to) 6.

In the diagram, ∠5 is adjacent to ∠EHD.

Angle

Angles are formed when two rays intersect at a point. An angle is also formed when to lines intersect each other, thereby the two lines share a common endpoint.

Adjacent angles are two angles that have a common side and a common vertex (corner point). They are placed side by side to each other.

In the diagram, ∠5 is adjacent to ∠EHD.

Find out more on Angle at: https://brainly.com/question/25770607

Diagnostic
Analytics

When completing an online shopping transaction, a typical shopper takes 7 seconds to
select each product and another 9 seconds to complete the check-out process. If it takes 37
seconds to complete a transaction, how many products are being purchased?
products
Submit

Answers

Answer:

In 26 seconds to complete a transaction, 2 products are being purchased.

Step-by-step explanation:

1 item  = 9 seconds

Time taken in all to check out = 8 seconds

Time taken to shop = 26 seconds

Now as check out process takes 8 seconds, so the

Time left to ACTUALLY SHOP =  Total Time  - Time Used to check out

                                                  = 26 seconds = 8 seconds = 18 seconds

Shopping of 1 item = 8 seconds

Shopping of 2 items  = 2 x ( Time taken in 1 item) = 2 x  9 = 18 seconds

So, in 18 seconds, 2 clothing item can be selected.

Hence,in 26 seconds to complete a transaction, 2 products are being purchased.

Find the value of x for which l||m

Answers

Answer:

30 =x

Step-by-step explanation:

The angles are correcting angle and corresponding angles are equal when the lines are parallel

55 = x+25

Subtract 25 from each side

55-25 = x+25

30 =x

Answer:

x = 30

Step-by-step explanation:

The 55 degree angle is congruent with the angle of x

55* = m∠x

Step 1. Take the equation of x (x + 25*) and line it up with 55

(x + 25) = 55

Step 2. Solve by subtracting both sides by 25

x = 30

find the angle measures given the figure is a rhombus.​

Answers

[tex] \large \tt{{❃ \: S \: O \: L \: U \: T \: I \: O \: N : }}[/tex]

A rhombus is a parallelogram in which all sides are equal i.e AB = BC = CD = CA

Let ∠ A be x. In the ∆ ABC , AB = AC which means they are isosceles triangle and we know the opposite angles of isosceles triangle are equal i.e ∠ A = ∠ C = x.

The sum of angles of a triangle is always 180°. Now , Find out the value of x :

[tex] \large{ \tt{❁ \:x + x + 98 = 180 \degree \: [ Sum\: of \: angle \: of \: a \: triangle ]}}[/tex]

[tex] \large{ \tt{⟶2x + 98 \degree= 180 \degree}}[/tex]

[tex] \large{ \tt{⟶ \: 2x = 180 \degree - 98 \degree}}[/tex]

[tex] \large{ \tt{⟶ \: 2x = 82 \degree}}[/tex]

[tex] \large{ \tt{ ⟶x = \frac{82 \degree}{2} }}[/tex]

[tex] \large{ \tt{⟶ \: x = 41 \degree}}[/tex]

The value of x is 41°. Now , Find the measure of ∠ 1 :

[tex] \large{ \tt{ ↔\angle \: 1 = x \degree = \boxed{41 \degree}}}[/tex] [ Being alternate angles ]

Hence , Our final answer is 41° .

- Alternate angles are the non-adjacent interiors pair of angles lying to the opposite side of a transversal when it intersects two straight line segments. Alternate angles form ' Z ' shape.

Hope I helped! Let me know if you have any questions regarding my answer. :)

Problem 1 Find the mBC.

Answers

Answer:

m BC = 100

Step-by-step explanation:

Since the angle is at the center, the arc has the same measurement as the angle

m BC = 100

Which is true about the solution to the system of inequalities shown?

y > 3x + 1

y < 3x – 3

Answers

they don't share any points

that's one thing, but I don't know what your options are of course.

see screenshot for illustration of the inequalities

How would I do this??

Answers

Part 1

[tex]\left(\frac{g}{h}\right)(x) = \frac{g(x)}{h(x)}\\\\\left(\frac{g}{h}\right)(x) = \frac{3x-5}{-2x^2+7}\\\\\left(\frac{g}{h}\right)(3) = \frac{3(3)-5}{-2(3)^2+7}\\\\\left(\frac{g}{h}\right)(3) = \frac{4}{-11}\\\\\left(\frac{g}{h}\right)(3) = -\frac{4}{11}\\\\[/tex]

Answer:  -4/11

====================================================

Part 2

Set the denominator function equal to zero and solve for x to find which values to kick out of the domain.

[tex]h(x) = 0\\\\-2x^2+7 = 0\\\\7 = 2x^2\\\\2x^2 = 7\\\\x^2 = 7/2\\\\x^2 = 3.5\\\\x = \sqrt{3.5} \ \text{ or } x = -\sqrt{3.5}\\\\[/tex]

This shows that if x is equal to either of those values, then the denominator h(x) will be zero. These are the values to kick out of the domain to prevent a division by zero error. Any other value of x is valid in the domain.

Answer: [tex]x = \sqrt{3.5} \text{ and } x = -\sqrt{3.5}\\\\[/tex]

Use the box plots comparing the number of males and number of females attending the latest superhero movie each day for a month to answer the questions.

Two box plots shown. The top one is labeled Males. Minimum at 0, Q1 at 3, median at 10, Q3 at 15, maximum at 35. The bottom box plot is labeled Females. Minimum at 0, Q1 at 2, median at 6, Q3 at 9, maximum at 14, and a point at 31.

Part A: Estimate the IQR for the males' data. (2 points)

Part B: Estimate the difference between the median values of each data set. (2 points)

Part C: Describe the distribution of the data and if the mean or median would be a better measure of center for each. (4 points)

Part D: Provide a possible reason for the outlier in the data set. (2 points)
PLEASE HELP ME!!! IVE BEEN AVOIDING THIS FOR SO LONG

Answers

(a) The IQR for the males' data is 12, because Q3 is at 15 and Q1 is at 3.

(b) The difference between the median values of each data set is 4 because 10-6 = 4

(c) For the males' data, the median would be a better measure of center, since the data is skewed right. For the females' data, the mean would be a better measure of center, because the data is pretty balanced (except for the outlier).

(d) Maybe a superhero movie had their opening night one day of this month, which would explain the outlier.

First, find the length of each edge of the cube.
Then, find the volume.

Answers

the length of each edge of the cube would be 6cm, and to find the volume you would do 6x6x6 which is 326 cm cubed

since in a cube length, width and height are equal,

h = 6

w = 6

l = 6

and volume is 6³ = 216cm³

hope it helps :)

A square has an area of 49 cm squared what is the length of each side

Answers

Answer:

7

Step-by-step explanation:

[tex]s = {a}^{2} \: thus \: a = \sqrt{s } = \sqrt{49} = 7[/tex]

The length of the of each side of square is 7

find cosØ if sinØ=-12/13 and tanØ>0.

A) -5/12
B) -5/13
C) 12/5
D) -13/12

Answers

Answer:

-5/13

Step-by-step explanation:

sin theta = opp / hyp

sin theta = -12 /13

we can find the adj side by using the pythagorean theorem

adj^2 + opp ^2 = hyp^2

adj^2 +(-12)^2 = 13^2

adj^2 +144 =169

adj^2 = 169-144

adj^2 = 25

Taking the square root of each side

adj = ±5

We know that it has to be negative since it is in the third quad

adj = -5

cos theta = adj / hyp

cos theta = -5/13

Answer:

B) -5/13

Step-by-step explanation:

i hope it will help

plzz mark as brainliest if you want

what is the sum of the square root of 63 and 144,divided by 5​

Answers

Answer:

Step-by-step explanation:

(√63 + 144)/5 = (3√7 + 144)/5 = 0.6√7 + 28.8

the answer is 3.98745078664 or ≈4

Calculate the mean of the following list of numbers: 10, 12, 7, 9, 9, 11, 15, 9
PLEASE HELP!!!!
9
9.5
10.67
10.25

Answers

Sun the numbers and divide by the number of numbers.
82/8=10.25
Other Questions
Write an equivalent expression for each of the following. Express your answers in the form a + bi or a - bi(5 + i)(9 + i)Help please What is the value of g? 14. What is the measure of angle w? Show all work. Find the angle that has tangent,1.000.Give your answer correct to two significant figures the velocity of a body is increases from 10 m/s ti 15 m/s in 5seconds calculate its acceleration 5 times a certain number plus 2 times that number plus 2 is 16 what is the number what is the best thing that has happened ? List at least two categories of comparison that could be used tocompare the arguments in Federalist No. 10 and Brutus No. 1. Between which longitudes does the region experience daytime? What name is given to a metal when it forms an ion, and what type of charge does it have? A car is traveling 40 kilometers per hour. What is the speed of that car in meters per second? Can any one solve this.Please During year 1 meriwerher construction company started a construction job with a contract price 3,000,000 the job was completed in year 2 Ideas for surrealist collage pls In the figure, p is parallel to s. Trasnversals t and w intersect at point L.StatementWhat is the missing reason in step 3?a.) Alternate interior angles along parallel lines are congruentb.) Alternate exterior angles along parallel lines are congruentc.) Corresponding angles along parallel lines are congruentd.) Vertical angles are congruent 1. If I meet him, I (tell)______ him to come see you immediatelychia cu iu kin loi 2 In a regression analysis involving 30 observations, the following estimated regression equation was obtained. If required enter negative values as negative numbers.In a regression analysis involving 30 observationsInterpret b1, b2, b3, and b4 in this estimated regression equation (to 1 decimal). Assume that for each coefficient statement, the remaining three variables are held constant. Enter negative values as negative numbers.b1 = estimated change in y per 1 unit change in x1 b2 = estimated change in y per 1 unit change in x2 b3 = estimated change in y per 1 unit change in x3 b4 = estimated change in y per 1 unit change in x4 Predict y when x1 = 10, x2 = 5, x3 = 1, and x4 = 2 (to 1 decimal). Which type of laws are passed by Congress and affect all states? federal lawsorstate laws Three construction companies have bid for a job. Max knows that the two companies with which he is competing have probabilities 1/3 and 1/9, respectively, of getting the job. What is the probability that Max will get the job? WHAT IS the side that is against the debate topic